Gems Junior 2 - AMTI

You might also like

Download as pdf
Download as pdf
You are on page 1of 225
THE ASSOCIATION OF MATHEMATICS TEACHERS OF INDIA GEMS From THE MATHEMTICS TEACHER JUNIOR -II Edited by Dr. M. Palanivasan THE ASSOCATION OF MATHEMATICS TEACHERS OF INDIA B-19, Vijay Avenue, 85/37, Venkatarangam Street, Triplicane, Chennai — 600 005. Tel: (044)-28441523 E-mail : amti@vsnl.com Web site: amtionline.com BETWEEN US Dear Reader, The demand for our books based on NMTC questions had been increasing steadily. Hence it is decided to bring all the problems and solutions of NMTC every five or ten years without disturbing the earlier editions, To satisfy the needs of various age groups they are being brought out in Jive different books — Primary, Sub-Junior, Junior, Inter ‘and RMO and INMO. The Present period covered by these books are from 2004 to 2012. In the case of RMO and INMO it is 2005 to 2013. We hope and trust the beneficiaries of our service will continue to encourage us to help aspirants of excellence in Mathematics Education. We are grateful to Smt. K. — Maheswari, Sri G. Gnanasundaram, Dr. M. Palanivasan and Sri V. Seshan respectively for having gone through the pages and edited these books in record time. With best wishes. Yours sincerely, (M. MAHADEVAN) General Secretary, AMTI CONTENTS (iivear2004 — Screening Test Final Test (2miyearj200: Screening Test Final Test ear2006 Screening Test Final Test Screening Test Final Test ff z Screening Test Final Test oo Be Screening Test Final Test v1 Screening Test Final Test Screening Test Final Test Screening Test Final Test 21 28 61 70 86 95 115 126 141 148 165 174 191 198 BHASKARA CONTEST SCREENING TEST - 2004 1. Given the sequence a,b, aab,cabb,aaabb, aaabbb,-»- upto 2004 terms, the total number of times a’s and b's are used from 1 to 2004 terms are (A) 2004 a’s and 2003 b’s (B) 4008 a’s and b's (©) 1002x 1003 a's and (1002)76’s (D) 1003? a’s and 1002x 1003 b’s Solution: Looking at the terms of the sequence given, it can be noted that all even members have the same number of a’s and b's while, all odd members have one ‘a’ more than the b's. Further in the n member, if n is even the number of a's = the number of b's= 3 and if n is odd, the number of a’s= #44, while the number of b's= 231. As 2004** member of the sequence corresponds to an even number 2004, the number of a's and the number of bs in this member will both be equal to 1002: As 2008 is odd, the number of a’s in the 2003" member is 203+! or 1002 while the number of b's will be 231 = 1001. ’. Number of a’s in the first 2004 members = 1414+2424+343-+4++++ 1002+ 1002 = 2(1-4 2-4 +4 1002) = 2% 1008 % 108 — 1900 x 1003, Also number of 6’s in the first 2004 numbers OF1F14242434+344+--- +1001 + 1002 2fL + 24-+--4 1001] + 1002 = 2% 1001 100? 4 so99 1002 x (1001 +1) = 10027, Answer: (C) i u 6 Problems and Solutions 2.. The number of two digit numbers divisible by the product of the digits is (A) 5 (B)8 (0) 14 (D) 33 Solution: Let the number be denoted by ab so that it is equal to 10a + b. As ab the product of the digits @ and 8, divides ! (10a +b), we must have ‘a’ as a divisor of b ie, a 0,y > 0 and.z > y. The factors Problems and Solutions 7 of zt—y4 are (zy), (a t+-y) and (xz? +y?). But these must be 1,3,5 in some order. Clearly c-y < uty <2? +y". Hence aw-y=le+y=8 and 2? +y? = 5, Therefore x =.2 and y= 1. Hence af +44 = ot 4 14 = 17. Answer: (A) 5. In this addition each letter represents a ABCD different digit. Which is the absent digit? “EBoD (A)1 (B) 3 (o)4 (D) 5 . : GHIJK Solution: Since the. problem involves nine distinct digits A,B, C,D,G, H,I,J and K,. only one of the digits from 0,1,2,3,:-+,9 is omitted.’ The sum. of two digits is another single digit or a double digited number with possibly carry over as 1. This implies that G = 1. But then A together with carry than 10. Hence A has to be 9.and the carry is 1 from hundred’s place addition. This forces H to be 0. Then I being distinct from H is not 0, Thus B.+ B plus a carry of 1 from the ten’s place addition will be greater than or equal to 11. Hence B > 5.1f B= 5, then J is either 0 or 1.. But already 0 and 1 are assigned to H and G respectively. Thus B is equal to 6,7 or 8 as 9 is already taken by A. The same argument shows that C and D Also cannot be equal to 5. The addition of D to itself gives the sum an even number in the unit’s.place, ie. K is even. So K # 5. Let us see whether J = 5. This is possible only 'if-C'= 2 or D > 5. If C = 7, then B= 6,D ='8 or B= 8,0 =6. lf B= 8, then D} 8. Similarly if B = 6 then K = 2. But then with a carry, from ten’s place I = 7, a digit oe ‘by C. This is not possible. Hence C # 7. Thus when: = 5,C can only be equal to 2. D has three: possible values are corresponding to which B has the-values 7 or 8, 6 or 8, 6 or 7 respectively. A solution exists only when'D = 7 and B = 8 » Problems and Solutions viz., 9827 + 827 = 10654. The digit 3 alone is missing in this equation. Note: We can have a few more solutions such as 9782 + 782 = 10564, 9728 + 728 = 10456 etc. But in all these cases also the digit 3 alone will be found missing. Answer: (B) Five children each owned a different number of rupees. The ratio of any one’s fortune to the fortune of every child poorer than himself was an integer. The combined fortune of the children was 847 rupees. The least number of rupees that a child had was (A) 12 Rs. (B) 10 Rs. (C)7Rs. (D) 5 Rs. Solution: Let the children have a,b,¢,d and e rupees such that they are integers anda >b>c>d>e. It is given that a+b+c+d+e = 847 and b,¢,d,e are divisors of a; c,d,e are divisors of b; d,e are divisors c; and e is a divisor of d. As e divides exactly a, b,¢,d and e, it is a factor ofa+b+e+ d+¢ = 847 =7x 11x11. In other words e is equal to 7 or 11 or 7x 11 or 11 x 11. But the least value obviously is 7. Hence any child must have had at least Rs.7. Note: A possible solution for a,b,c, d,e will bea=b=c=d= 210 and e = 7. Answer: (C) . A number with 8 digits is a multiple of 73 and also a multiple: of 137. The second digit from the left equals 7. Then the 6" digit from the left equals (A)L (B)7 (C)9 + (D) can be any digit Solution: The number is a multiple of 137 and 73. As 137 and 73 are primes, the number is also a multiple of 187%73 = 10001. ‘The eight digit number which is a multiple of 10001 can be got Problems and Solutions 9 by multiplying a four digit number abed by 10001. The product is obviously abed abed. ‘The second and the sixth digits from the left are both equal to b. Hence if the second digit is 7 then the sixth digit is also 7. Answer: (B) 8. Let n be the least positive integer such that 1260n is the cube of a natural number. Then n satisfies (A)l 0 or (n42)? $9. Hence »+.2 = —3,~-2,-1,0,1,2,3. As nis an integer, n can. be —5,—4,—3,—2,-1,0 or 1. Hence there are seven possible values for n. Answer: (C) 12, During holidays, five people A, B,C, D and E went swimming regularly. Each time they went, exactly one of them was missing. A went the least number of’times (5 times)-and 2 Problems and Solutions 1 most often (8 times). What can we say about the number of times B,C and D went? (A) each went six times (B) each went seven times (C) 2 went 6 times and one went 7 times (D) 2 went 7 times and 1 went 6 times Solution: From the given information we observe that (i) E was present on all occasions, (ii) A missed out three occasions, (iii) B,C, .D together missed 5 occasions and (iv) no one except A missed 3 or more occasions. We, therefore, infer two of B,C, D missed 2 occasions while the third one.occasion. In other words among B,C,D two went swimming on 6 occasions and the other went on 7 occasions. Answer: (C) 13. Let A = {a,b,c} and B = {a,b,d,e, f}. How many sete C consisting of characters from the English alphabet can be constructed so that CC Band such that ANC has one element and CGA. (A) 16 (B) 14 (C)8 (D) 6 Solution: From the given data we infer that C is part of B but not of A and C and A have just one element in common. As ANB = {6} and C is a part of B, ANC also has the element b. Hence C is the set containing b and elements of any subset of {a,c,d,e}. As {a,c,d,e} has 24 = 16 subsets, we can form C in 16 different: ways. Answer: (A) 14, The sum of all angles except one of a convex polygon is 2190°. (where the angles are less than 180°). Then the possible number of sides of the polygon is (A) 13 (B) 16 (©) 17 (D) 19 12 Problems and Solutions Solution: If the convex polygon has n sides, we are given: thet the sum of the angles of the polygon except one = 2190° = 12 x 180° + 30°, As the sum of all the n interior angles is (2n—4) right angles or. (2n—4) x £ = (n—2) x 180°, the angle left out must be 150°, the supplement of 30°. Then sum of the interior angles = 13 x 180°. n=2=1orn=15. Hence the given: polygon has 15 sides. Answer: (B) 15. In aright angled triangle with legs 4 and 8 , the area of the largest square that can be inscribed in the triangle is: (A) § ) 4 )¥# ©) F Solution: Let the square A inscribed be one with side a. tT Then iy Ne area AABC'= area. AAFE + 2 area AEDC 45> + area of square BDEF ie, px 4x8 = Me 4 Md +o ie, 32=1200ra= 9. Hence the side of the inscribed square = §. Answer: (A) 16. Two circles with centers A and B and radius 2 touch each other externally at C. A third circle with center C and radius 2 meets the other two at D, B (ses the figure).. Then area ABDE ig (A) 3v2 (B) 6v3 (©) 3v5 (6) 6v3 Problems and Solutions 13 Solution: -Clearly AAEC and ACDB are both equilateral and congruent with ‘sides equal to 2 unitd. ADCE is also ‘an equilateral triahgle with ‘each side equal to 2 units. So, ED = 2 unite, But AB = “AC + CB = 4 units. Area of thé quadrilateral ABDE ‘(in fact,. this-is a trapezium) ia’ equal to sum of the areas of the identical equilateral triangles AGE, ECD-and CBD of sides equal to 2 ‘units. Hence, the required area = 3 x } x (48 x 2) x 2= 3V3 square units. . ‘ . " Answer: (C) 17. In AABC, ZA =.90° and J is the inicentre: ‘The perpendicular distance of I from BC is ./8.' Then AT is equal to (A) v8 (B)3 (C). viz (0) 4 Solution: Let the incirdle touch the sides: BC, CA; AB in P,Q,R respectively. “As m£BAC = 90°, AQIR is a square of side = IR'= in redius =r, say. As IP =r= V8, (given), AI the diagonal of the ‘square AQIR is IRx V2 = V8v2 = 4 units. Answer: (D) 18. In an isosceles triangle, ‘the centriod, the ortliocentre, the incentré eid the circumcentre are (A) conincident (8) collinear (C) in the ‘interior of the circumcircle (D) it the interior of the incircle Solution: AABC is isosceles with ZB = ZC. Let AD be the altitude from A. The orthocentre H leg on AD,.Also AD 14 Problems and Solutions bisects the vertical angle BAC and so it contains the incentre I, Further AD bisects perpendicularly BC. ‘Hence it contains the circumcentre S and being a median it also contains thé centroid G. Hence H,I,§ and G are collinear, Answer: (B) Note: These four points will coincide when’ AABC' is equilateral. When A is obtuse H lies outside the circumcircle as well s the incircle. Thus (B) is the only correct answer. 19. If a,b are positive real numbers and \/a¥ = a../§ where af is a mixed fraction, which of the following is true? (A) b=0? +1 (B)a=-1 (C)a=P4+1 (D) b= Solution: The value of the mixed fraction af is a+ $. ‘Therefore the given condition implies. 2-1 a _ 2,2, ab+1) _ a ot =. Ke Ee b+1=a7 (Cancelling $ on both sides, a > 0,b > 0). Answer: (D) 20,12 +E+E= land $+$+£ =O then the value of B+ 5+ is (A) 0 (B) -11 (c)9 @)1 Solution: Writing 2 = atytz=landit3+ yz +22 + cy = 0, Now, = Gand z = , we are given that =0. The second equation implies atyte? = (2+y+2)?—Ayetect ay) = P-2x0=1 Wl i Pe = = 1, Answer: ie Gtyta =) (D) Problems and Solutions 15 21. Let [2] denote the greatest integer integer less than or equal to x, what is the value of [VT] + [V2] + [v3] +--+ + [V2004]7 (A) 58850 (B) 59730 (C) 59950 (D) 56718 Solution: We have (m+ 1)? = m? + 2m +1 so that the integer parts of Vmi2, Vm? + 1, Vm? +-2,°-- , Vm? + 2m will all be m so that [Vm] + (vm? +1) +--+ (Vvm™+2m = (2m + 1).m = 2m? +m. Also 2004 = 1936 + 68 = 447 + 68. [V44] + (VEd7 41] + «+: + [VEE 6B] = 69 x 44 Hence the value of the required sum 43 ; = Lian) + (Vint 1] +--+ [Vn + Bm + 69 x 44 mat 3 = Lem? +m) +69 x44 mel i aha9(es +3y(2 x 4343) BA go x a4 = 43x 44% 294 43 x 224+ 69 x 44 = 22x [2404 +43 +138] = 22 x 2675 = 58, 850. Answer : (A) 22. If the roots of the equation x? —2ar +-a?+a—3 =0 are real and less than 3 then (A)a<2 (B)24 Solution: From the given equation, we get (x ~ a)? = 3—a or =a+ /3—a, As the roots are real 3a > Oora <3. As the roots are also less than 3, we havea + /3—a <3 and a-V3—a < 3.From a+ V3—a < 3, we get S-a <3—a This implies « # 3. Hence a < 3. As 3—a can be written as vV3~aV3—a and Y3—a > 0, we get 1 < V3—a. Hence a is not greater than or equal to 2. In other words a < 2. When @<2,a— /3—a <3 is certainly true. Answer: (A) 16 Problems and Solutions 23. If a function f(z) is defined such that 10/).= 152 where z <1, then f(a) + f(6) is equal to (A) £(288) (8) £( #88): (©) F(#5%) (D) none of these Solution: f(a) and f(b) are defined only ifa <1,b< 1. 1-b FC airs (b) ia 197) & and 19/ 7 Hence tof@+s0) _ (-a)(1~) +a) (1+) (1+95)—(o+b) _1~(AH5) _i-2 = (¥ab)+ (ats) Ee Cte) © iy = 10/) where 2 = =f =, provided x’< 1. Now, c= ft4< Lifl+ab>a+b i.e. if (1 ~a)(1— 6) >.0, which is true ds a < 1,0 <1. F(a) + F(0) = #(2) = FS). ‘Answer! (A) 24. How many solutions are there for (a,8) if 7a073 is a five digit number divisible ‘by 997 8 (A)3 - (B)2 (C) 0. (D)1 Solution: We can write 7ab73 as 7ab hundreds.+73. But this is same as 7ab times 99 plus 7ab + 73. For 99 to be a divisor of 70673, we therefore have 7ab+ 73 a8 a multiple of 99. ” Giving various digital values to a,b formn 0, 1,2,+++ ,9, we find that 7ab+ 73 can be a number from 700+73 to 799+73 i.e, & Problems and Solutions 17 nuniber from 773 to 872. Ini this range only 792 is divisible by 99. So, 7ab coincides with 792 — 73 i.e., with 719. Therefore, a = 1,6 = 9. Thus there is only one pair of values (1,9) for (a,b). Answer: (D) 25. _The number 107% — 76™ is divisible by (A) 61 (B) 62 (C) 64° (D) none of these Solution: If-a,b (a > 6) are real numbers a” —0” is divisible by a—b for integral n > 1. Now 107% —-76% = (1077)* — (767)® is divisible by 107? — 76? i.e., by (107 + 76)(107 ~ 76) i.e., by 183 and 31. i.¢., by 3,31 and 61. The option (A) contains 61. ‘Answer: (A) 26. A sequence a, a1, 42,43," ,@n-*+ is defined such that a9 = a, = 1 and any; = (ap-1.0) + 1'for n > 1. Which of the following is true? (A) 442000 — (B) 3fa2008 © (C) Blazons = (D) 2 F a2n03 Solution: We have ani1 = On-dn1+1, a9 = a = 1. 1,02 = 2, ag =3, a4 =7, a5 = 22, ag = 155, a7 = 3411,--- Expressed congruent modulo 4 the sequence is 1,1,2,3,3,2,3,3,--- This is a cyclic sequence from a2 onwards, the numbers 2,3,3 cyclically repeating. Therefore ag994 = 3(mod 4). So, 4 does not: divide agoo4. (If it is a divisor, ago9q must be congruent zero modulo 4). Similarly expressing the sequence congruent modulo 2, modulo 3 and modulo 5, we get respectively (i) 1,1,0,1,1,0,1,1,--» congruent: modulo 2 (fi) 1,1,2,0,1,1,2,0,-+» congruent inodulo 3 (iii) 1,1, 2, 3, 2, 2,0, 1,1, 2,3, 2,2,0,--- congruent modulo 5. From (i) 2903 = 0(mod 2). _.*, 2 divides ago03- From (ii) a2o03 = 0(mod 3). ..3 divides az003. 18 Problems and Solutions From (iii) ag904 = 2(mod 6)... 5 does not divide ago04 exactly. Thus (A) alone is true. “Answer: (A) 27. A solid cuboid has edges of length a, 6, c. What is the surface area? (A) (a+b+c)? — (a2 +0? +c?) (D) ab + be-+ ca Solution: Surface area of the cuboid (from the figure) = 2(ab+ be+ca) = (a+b+c)?— (a? +74?) Answer: (A) (B) abe (C) 2a? +82 +2) a 28. A circle and # parabola are drawn on a piece of paper. The number of regions they divide the paper into is at most (A)3 (B) 4 Solution: The parabola can cut the circle in a maximum of four points. When the parabola does not touch the boundary of the paper as in the adjoining figure, we get the number of regions as 6. However when the boundary of the paper touches the parabola region II of the figure is further divided into 2 regions - II(a) and II(b). Thus the maximum number of regions got is 7. (C)5 (D) 6 Note: The options do riot contain the number 7, but have only Problems and Solutions 19 the number 6. Assuming that the paper is sufficiently large that the boundary is not a tangent to the parabola, option (D) is correct. 29, A-cubic polynomial P is such that P(1) = 1,P(2) = 2, P(3) =3 and P(4) = 5, Then P(6) is (A)7 (B) 10 (©) 13 (D) 16 Solution: Let p(z) =a+ bx +er?+dz* Then =a+b+ct+d (i) a+2b+4e+8d ° (ii) +3b+9c+27d- (iii) a+4b+16¢+ 64d (iv) gives 1=b+3c+7d (v) ) gives 1=b+5e+19d (vi) iii) gives 2= 6+ 7c+37d (vii) (vi)-(v ) gives “O=2c4 12d (viii) (vii )—(vi) gives 1=2c+18d (ix) (ix)—(vili) gives 1=6dord=} .. (ix) gives, 1 = 2c+3 0re= . (v) gives —3+h orb= ¥. 2. () gives l=a4+%-14§ ora=-1. Hence p(x) = -1+ ¥a— 2? + d28, o. p(6) = 1+ % x 6-36 + § x 216 =-1417- 36436 = 16. Answer: (D) 30. Which of the following ia the best approximation to (28 =1)(38 — 1)(4 - 1) +++ (10008 ? (2+ 1)(85 + 1)(48 + 1)--- (2000 +1) (a)} (3) ¥ (©) #8 (0) 88 20 Problems and Solutions Solution: Note that m—1 (m—1)(m? +m #1) m+1 = (m+1)(m?—m+4t)) m+m+l = (mt1)?-(m+1)+1. . Given expression As 2+24+1=3?-341,97+3+1=4—4+ 1 etc, after cancellations, the given expression = 3-2-3 + 29 4adeeaonoe1 1001001 _ 333667 __ 2x (1000000 + 1000+ 1) 1001000 ~ 500500 2 =——“To0 x toolx3 = 3 * = 0.666001932 ‘The given options are equivalent to 0.6, 0.66. 0.666 and 0.6666. Hence the expression is nearest to 0.666. Answer: (C) Amicable Numbers ‘Two natural numbers im and n are called Amicable (or friendly) if m is equal to o(n) and n is equal to o(m) where o(k) denotes the sum of all the proper divisors of k. For example, 220 and 284 are amicable numbers. If two numbers are Harshad numbers and, are also amicable, they are called Amicable Hershad numbers. (2620, 2924) is one such pair. The Wonder World of Kaprekar Numbers ~ Page No.68-69. BHASKARA CONTEST FINAL TEST - 2004 1. Show that there are no integers a,b,c for which a? +P -8c=6 Solution: For any integer n we always have w= 01,4 (mod 8). So the’ different possibilities for a? + 6 modulo 8 are O+0=0, OF1=1, 14+1=2, 144-5, 044=4, 444=0 But a? +5? — 8c = 6 implies that a? +b? = 6(mod 8) which is impossible as per the above observation. Hence there are no integers a, b,c for which a? + 0? ~ 8c = 6. 2. Given that N = 2"(2"+1 — 1) and 2°*? —' is a prime number, show that a) Sum of the divisors of N is 2N b) Sum of the reciprocals of the divisors of. N is 2. Solution: Let 2*+!-1= P. Then we are given that Pisa prime number. We have 2"? = P+1 and 2" = 2}. This makes N = 2" x P, We find that the divisors of 2" x P are 1,2,2,28,.-» 2", P, 2P,2°P,29P, ++» ,2"P. So the sum of the divisors of NV is given by on = Lede epee ao FP(LE 242429406642") (P+1) BS *) (P+)@"-1) = oantt—1) = IN This proves (a). Ml au 21 22 Problems and Solutions Remark: One can also use the formula for the sum of the divisors on of a positive integer n given by n= peti) (pitt (3 ~ (@) " pai on “pot Prod where n = phipk? ...pit...pkr is the prime factorization of n. vax —_ Pi-1 2-1 P-1 (2"*1 —1)(P +1) 2-an(gntt — 1) = 2N, a oN Al We have sum of tne aesprece® of “e Gvisors ot Nas fe = GAEt ae tert 2 # sits to r+: to Ay tet aee +3 wlu+d 1-(y) (p41 ( i-} ) x e) (art! —1) grt a1) * GTI = 2 This proves part (b). 3, Given three non-collinear points A, B, C' construct a circle with center C' such that the tangents from A and B to the circle are parallel. Solution: Construction: Find the mid-point M of AB. Join C to M and draw lines AX and BY parallel to MC. From C Problems and Solutions 23 > drop perpendiculars to AX and BY to meet them at P and Q respectively. Draw the circle > with center C and radius CP, ¥ is the required circle. Proof of Construction: Since AM = MB and AX||MC||BQ we have O'P = CQ. Now AX and BY are parallel tangents to the circle >. Given a circle with diameter AB and a point X on the circle different from A and B, let ta, ty and tz be the tangents to the circle at A,B and X respectively. Let Z be the point where the line AX meets tb and Y be the point where the line BX meets ty. Show that the three lines YZ,t- and AB are either concurrent or parallel. » Solution: 24 Problems and Solutions As AB is a diameter, we have ZAXB = 90°. Let LXAB = 0 and 2XBA = ¢. We note from the “angle in the alternate segment” theorem that LXAP = 9, LAXP = $= ZZXQ, 2BXQ=0=ZYXP ZAYB = 6, ZAZB = $,£XBQ=0 In the right angled triangle AXY, we have PA = PX = PY. Therefore, P is the mid-point of AY. Similarly from ABXZ we get QX = QB = QZ, Therefore Q is the midpoint of BZ. Also AY is parallel to BZ. Hence YZ, PQ, AB concur at a point. If X is the mid-point of the arc AB, we see that the lines Y Z, PQ, AB are parallel. 5. The polynomial ax? + bc? + cz +d has integral coefficients a,b, c,d. If ad is odd and bc is even show that at least one roat of the polynomial is irrational. Solution: Suppose the roots 21, 72,23 of az + ba? +er+d=0 are all rational numbers. Then the transformation y = az transforms the given cubic into of) +0(E) +e(G This is equivalent to y+ by? +acy+a%d=0. q) “The ronts of this equation are +d=0. Vi = G21, y2 = GLa, y3 = as. Problems and Solutions 25 It is clear that the roots y1,42,y3 are all rational numbers. ‘As the coefficients of equation (1) are integers, we must have Yi, ya,y3 08 integers. Further these integers yi, y2, v3 satisfy nutytys=—b (2) giya + yays + yay = ac (3) uivays = —07d. (4) Now, in view of equation (4), v1, ¥2,s must be divisors of a7d. Since it is given thet ad is an odd integer, 41, ¥2, ys are all odd integers. From equations (2) and (3), it is clear that 6 and ac must be odd. This means thet a,b,d are all odd integers. ‘Therefore, equation (3) implies that c is an odd integer. Now, b and c are odd contradicts our hypothesis that bc is even. So the given equation has at least one irrational root. 6. Let f be a function from N to R satisfying (a) f(1) =1 and (b) f(1) + 2f(2) +3f(8) +--+ afm) = n(n + YF (n). Find f (2004). Solution: We have £(1) + 2f(2) + 8f (3) +--+ + nF (n) = a(n + LF) f(1) +2F (2) +3f (8) + +++ + (n-1)f(n—1) = (n= 1)nf(n—1) for n > 3,. Then subtracting the second equation from the first, we get n? f(n) = n(n—1)f(n—1) or (n=) p(n 1) =nf{n), forn>3 2f(2) = 8f(8) = 4f(4) = +++ = nf (n) for n > 2. 26 Problems and Solutions From (b) with n = 2004, we get F(1) + 2f(2) + +++ + 2003. (2003) = (2004)? £(2004) * 1+ (2002) x (2004) F(2004) = (2004)*f(2004) 1 3004(2004 — 2002) 1 4008 ©. #(2004) = 7. Consider a permutation pipopapupsps of the six numbers. 1,2,3,4,5,6 which can be transformed to 123456 by transposing two numbers exactly four times. By a transposition we mean an interchange of two places — for example, 1 2 3 4 5 6 to 3214 & 6 (positions 1 and 3 are interchanged). Find the number of such permutations. Solution: We observe that any permutation of {1,2,3, 4,5, 6} can be reached in at most 5 transpositions, For any permutation P given by pipopapapsps we count the number of pairs ¢ < j such that p > pj and call it c(P). Then every transposition applied to P changes the (odd-even) parity of c(P). This means that the set of all permutations can be separated into those reachable by an odd number of trenspositions and those reachable by an even number of transpositions. Any permutation reachable in 0 or 2 steps can be algo be reached in four by repeating the same transposition twice. So we are just required to count the number of even permutations. Fix a transposition T. Then T defines a correspondence P ~» TP from the set of even permutations to the et of odd permutations, which is clearly one-one. This means that the number of even permutations is one half of the Problems and Solutions 27 total number of permutations, which is 6! = 720 in our case. So the answer to our question is 360. 8. Let 01,02,03,'-* ;@m be a sequetice of real numbers, The sum of k— successive terms is called a k— sum, for example Oj +0j41+;40+++++0;44-1 18 a k— sum. In a finite sequence of real numbers every 7-sum is negative and every 11-sum is ypositive. Find the largest number of terms in such a sequence. “Solution: We write the successive 7-sums in separate rows: If we continue writing till the first 11 such 7-sums, we get 11 rows of 7-sums; which in turn gives 7 columns’of 11-sums. See the adjoining table. ay +og+03+++-+ar < 0 agt+a3+aa+-s-+as < 0 agtag+a5t---+ag < 0 <0 , Writing n in 6" as r (mod 5). We find that the last two digits are 36,16,96,76 and 56 respectively when r= 2,3,4,0 and 1. ‘As 2005 = 0{ mod 5), the last two digits of 679 are 76. Hence (2006)205 also has last two digits as 76. Answer: (C) 11. A cone is made from a circular sector, by joining the two radii and the ratio of the radius and slant height of the tear cone is as 1:2. Then the angle of the sector from which the cone is made Is (A) 90° (B) 35° (G) 180° (D) 70° Solution: Let r be the radius of the base circle of the cone. Then the slant height is 2r. But this must be the length of the bounding radii of the sector from which the cone is made. Hence ‘if the angle of this sector is @, then the bounding arc of the sector 2r@ is also equal to the circumference 2nr of the base circle of the cone. Thus 2r8 = 2ar or @ = 7.= 180°. Answer: (C) * Aliter: : If R is the radius of the circle and 9 be the angle of the sector, then /2 is the radius of the base circle of the cone. Noting that the area of the sector is the area of the curved surface of the cone we have $R?0=7.8+R.Hence @= 4. Answer: (C) 12, The number of prime’ numbers less than 100 which can be expressed as the sum of the squares of two natural numbers is au (B) 12 (C) 16 (D) 20 54 Problems and Solutions Solution; Enumerating the numbers 2 = 1+1, 6 =1+4, 13 = 449, 17=1+416, 29=4+ 25, 37=1+436, 41 = 16+ 26, 53 = 4449, 61 = 25436, 89 = 25+ 64 and 97= 16481, we find that there are 11 such numbers. Answer; (A) 13. Given the equation of the circle x? +? = 100, the number: of points (a,4) lying on the circle, where ‘a’ and ‘b’ are both integers is (A) 2 (B) 4 (C)8 (D) 12 Solution: The contre of the circle-is (0,0) and radius is 10. When « = 0,y = +10 and when z = +10,y = 0. Thus (0,410) and (+10,0) are 4 points satisfying the conditions of the problem. Further when 2 = +6, we have y = +8 and when z= £8 we have y= 46. Thus (+6, +8) and (8,46), totally 8 points are also on the circle satisfying the given conditions. Apart from these twelve points for no integral values of z can ‘we get integral values of y satisfying x* + y? = 100. ‘Thus (a,b) -has twelve solutions. Answer: (D) 14. In the adjoining figure P,M,Q and RF .are collinear points are PM = MQ = MS. It is also given SR? = PR.QR. Then (A) ZQ9R = ZMSP* (B) ZQSR = ZMsQ (C) ZQSM = PSM (D) 29QR = ZSMP Solution: As PM = MQ = MS, the point M is the circumcentre of APSQ. Problems and Solutions 35 As RQ.RP = RS?, the tangent from R touches this circle at S. Hence ZQSR = angle in the alternate segment 2SPM = ZMSP (- MS = MP). Answer: (A) 15. Seven consecutive numbers are chosen. From these seven numbers, two numbers are chosen. What is the probability that the sum of the two numbers chosen is divisible by 7? (A) } 4B) 3 (©) 3 ) 4 Solution: Expressing the seven. consecutive numbers in the form r( mod 7), we have for r the values 0,1,2,---6. The numbers leaving remainders (1,6); (2,8) or (3,4) when added up will give remainder 0 when divided by 7. ‘Two numbers out of séven'can be chosen in 7C2 or 146 or 21 ways and of these there are 3 ways favourable for the compliance of the required conditions. Hence the probability is 3/21 = }.. Answer: (A) 16. If two successive discounts of a% and b% are given on the sale of a certain article, the single equivalent discount is (A)o+t+3 = (B) a+b- (C) o-b+ yy (D) ab - Solution: If 100 is the marked price, after a discount of 0%, the sale price is 100-a. After allowing another 6% the sale price will be (100 —.o)(1 — yf) = 100-a—b+ gh. , Effective equivalent discount is 100 — [100 —a@—6+ 2) ie, atb—-.. Answer: (B) 17. Ini a triangle ABO, ZA = 30°, BO = 6m. Then the radius of the circum circle is (A) 3em (B) Gem (C) 8cm (D) 4.5em 36 Problems and Solutions Solution: If A 1s the radius of the circum circle then WR= PF.» R= 8S = zabgs = Gem. Answer: (B) 18. AB is the diameter of the circle. Then 2QPB is (A) 40° (B) 50° (C TD (A) 40° (B) 50° (C) 60° (D) 30° f ; 7 Solution: In the question ZQBP umust be changed into ZQPB. From the right angled AAQB, . ZQAB=90°-ZQBA =90° — 40° = 50°. But QAB = ZQPB (angles is the same segment) ZQPB-= 50°. Answer: (B) Note: If, as the figure suggests, PQ is perpendicular to AB, then ZPBA =.ZABQ = 40° 80 that ZQPB = 80°. This answer is not available in the choices given. 19. If Fe w+ st where w,x,y are integers then w-+o+y 3 equals (A) 16 (B) 17 (c) 18 (D) 19 Solution: As w+ sty = ff=5+%, 7 equating the integral and fractional parts on both sides, we get w=5 and air = ast = & implies t= B or c+ h=9+}. Hence z=9 and }=} or a=9y=2, Thus w=5,0=9,y=2 so that w+aty=5+9+2+16, Answer: (A) Problems and Solutions 37 20. PQRS is a common diameter of the three circles, ‘The area of the middle circle is the average of the areas of the other two, If PQ = 2 RS =1 then the length of QR is (A) 14+-V8 (8) Vé~1 (0) 4 (D)3 Solution: Let diameters PQ,PR and PS be D;,D2,D3 respectively, © Then 772 = > 5 (Di + D3)] > 2D} = D3 + Di. Given D, = PQ =1;D3— Do =RS=0.5 aD} a 2(D3—0.5)?=14+D3 or D3-2Ds~0.5=0. 2. Ds = YE 14 8. Length QR = D,—D, = (D3—-0.5)—1= Dg—1.5 = -a¥8, As QR is not negative, QR = =y¥8, Note: None of the given options gives the answer Answer: (=15¥8) . 21. The positive numbers z and y satisfy zy =1. The minimum value of 2+ gir is (A) (B) 3 (c)1 (D) no minimum ‘Solution: The problem is to minkmize f = 2y + gly subject to 2y=1 or to minimize f =2~* + 2, For minimum f, £ =0 = —da-$ 4.29 > 24 =4 or 24 =2. Also £4 = 202-5 +32? > 0 for z>0. «. f has a minimum at the positive 4th roots of 2. , Minimum value = $4+44%2=1. Answer: (C) 58 Problems and Solutions 22. Of the points (0,0), (2,0), (3,1), (1,2), (3,3), (4,3) and (2,4) at most how many can be on a circle? ‘ays (B) 3 (c)4 (0) 6 Solution; Denoting the seven given points by O, B,C, D, E, F and G, from the rough sketch we see that all the points except F are at the same distance V5 units from D. (Note that if the figure represents a chess board “a horse” placed at D can in a single move go to O,B,C,E or G@ but not to F). Thus the five points 0,B,C,E,G are concyclic. The equation to this circle is (x — 1)? + (y-27 =5. Answer: (A) 23. How many of these expressions 2° + y4,2* + y3,23+y3 and x*—y‘ are positive for all possible numbers z and y for which z>y?. (A) i (B) 0 (G)2 (D) 3 Solution: Construct the following table: (x > y). aly [ete [ety [ety lee] Oof-} + - - - -fep + + = 7 +t-] + + + + +4] + + + + +40 + + + | + From this table one can see that for all values of 2,y with z>y, only 234 is positive. Answer: (A) 24. The number of ordered pair of digits (A,B) such tha A 3640548981270644 B is divisible by 99 is ~ (A)3 (B)2 {C) 1 (D) zero Problems and Solutions 39 Solution: As the number is to be divisible by 9 and 11,. (i) sum of the digits is divisible by 9, and (ii) difference between the sum of the odd digits and the sum of the even digits is-a multiple of 11. ‘Thus we have 71+ A+ B is a multiple of 9.or A+ B—1-is a multiple of 9 and (37 + A) ~ (34+ 8) is divisible by 11. This A+B = 1,10 or 19 and A~B = ~—3. Obviously A+B #19 as A and B are single digits. If A or B is aero then the other can be one but the difference will not be 3. The remaining possibility is that A+ B=10 and A-B=—3. ‘This is also not possible since if A+B is even, then A—B must also be even, No values of A and # exist. Answer: (D) 25, The roots of the equation 2°—40x‘+P25+Q2?4+Rr+5=0 are in geometric progression. The sum of their reciprocals is 10. Then [S| is equal to (A) 16 (B) 32 (c)4 @)1 Solution: Let the roots be ar*,ar,a, 2,3. Then sum of the roots = a(r?+r+1+1+4 4) =40. The sum of their reciprocals = 14 +14+1+r+r?) = 10 dividing one by the other a? = 4 or a= 42. Product of the roots =.a5 = (+2)§ = 432 =-S. [S| = 32. + Answer: (B) BHASKARA CONTEST FINAL TEST - 2005 1. p(z) is a polynomial of degree n and p(k))= wiz for k=0,1,2,--+,n, Determine p(n +1). Solution: We observe that (k + 1)p(k) — k = 0 for all k in {0,1,2,---n}. So we consider the polynomial q(z) = (2 +1)p(z) — 2 and we see that the polynomial q(z) has the following n+1 numbers 0,1,2,-++ ,n as roots. Therefore 9(2) = Ka(z -1)(z—2)(e-3)...(~n) for some-constant K . We evaluate q(x) at —1 to get (-1) = ~(-1) 4 = K(-1)(-2)(-3)--- (-n-1) = K(-1)"*" (m4)! wir i Thus K = (-1)' aa As p(x) = ate) +2, we get +1 _ an+)t+(nt)) n= att QL + Yin)(n—1yin— ~ nt2 _ ntit(-2)t ~ n+2 Therefore, 1 ifm isodd p(n +1) = ifn is even nt+2 40 Problems and Solutions 41 2, Two sides of a triangle are V3cms and V2cms. The medians to these sides are perpendicular to each other. Find the third side. Solution: Let AD,BE and GF be the medians through the vertices A,B,C - of = AABC. Let BE and CF E be the given perpendicular medians. Using cosine formula B c AB? = AD? + BD? — 2AB.BDcos ADB. and AC? = AD? + DC? — 2AD.DC cos ADC. But cosADC =—cosADB and BD= DC = ize. Hence adding, we get AB? +AC? = 2AD? +2(2 oe 2. 5B). <. 4AD? = 24B? + 2AC? — BC. Similarly 4BE? = 2BC? +2AB? — AC? and ACF? = 240? + 2BC? — AB. As G divides each median in the ratio 2:1 we get BG? = Spe? = tan? +280? - CA), OG? = S08? = HAC" +2042 — AB), Since BEC = 90°, BG? + CG? = BG”. v Gl@AB? + 280? — CA?) + + (280? + 20.8? — AB) = BC?. or 2AB? + 2BC? ~ CA? +2BC? + 20.4? — AB? = 9BC*

You might also like